Supporters MGMAT RC

This topic has expert replies
Legendary Member
Posts: 809
Joined: Wed Mar 24, 2010 10:10 pm
Thanked: 50 times
Followed by:4 members

Supporters MGMAT RC

by akhpad » Sat May 15, 2010 5:29 am
Source: Manhattan GMAT CR Guide 4th Ed
-----------------------------------------------------

Supporters of a costly new defense advanced research projects Agency (DARPA) initiative assert that the project will benefit industrial companies as well as military itself. In many instances, military research has resulted in technologies that have fueled corporate development and growth, and this pattern can be expected to continue.

Each of the following, if true, serves to weaken the argument above EXCEPT:

A: The research initiative will occupy many talented scientists, many of whom would otherwise have worked for private corporations.
B: In past decades, DARPA has adopted an increasingly restrictive stance regarding the use of intellectual property resulting from its research.
C: A high proportion of the government resources directed towards the initiative would ordinarily have gone to tax subsidies for various business.
D: The research initiative is focused on materials development through nano-technology, which is thought to have many commercial applications.
E: The DARPA research makes use of manufacturing processes the would be cost-prohibitive for most companies to replicate.

OA: D

I understood stimulus but Logically, I could not understood any of the choices that how they strengthen or weaken.

Same problem is also in image.You can refer image if there is any typing mistake.

Image

Master | Next Rank: 500 Posts
Posts: 134
Joined: Mon Mar 01, 2010 7:08 am
Thanked: 3 times

by bupbebeo » Sat May 15, 2010 5:51 am
akhp77 wrote:Source: Manhattan GMAT CR Guide 4th Ed
-----------------------------------------------------

Supporters of a costly new defense advanced research projects Agency (DARPA) initiative assert that the project will benefit industrial companies as well as military itself. In many instances, military research has resulted in technologies that have fueled corporate development and growth, and this pattern can be expected to continue.

Each of the following, if true, serves to weaken the argument above EXCEPT:

A: The research initiative will occupy many talented scientists, many of whom would otherwise have worked for private corporations.
B: In past decades, DARPA has adopted an increasingly restrictive stance regarding the use of intellectual property resulting from its research.
C: A high proportion of the government resources directed towards the initiative would ordinarily have gone to tax subsidies for various business.
D: The research initiative is focused on materials development through nano-technology, which is thought to have many commercial applications.
E: The DARPA research makes use of manufacturing processes the would be cost-prohibitive for most companies to replicate.

OA: D

I understood stimulus but Logically, I could not understood any of the choices that how they strengthen or weaken.

Same problem is also in image.You can refer image if there is any typing mistake.

Image
Hey akhp77,


let's consider all answer choice

A, the initiative will occupy talented people. how corporation will develop if they do not have talented people. therefore, this answer choice weaken the argument.

B, if the initiative prevents corporation from approaching their research results. How these companies can get benefit from the initiative

C, subsidies for companies will be lost because resoures for initiative will be drawn from these subsidies

D, this will support the argument. therefore, it is the correct answer. Nano technology has many commercial application. therefore, it helps companies.

E, if the initiative uses technologies which are too expensive for companies to apply. What benefit companies can get from application of prohibitive technologies. therefore, it weaken the argument.


out of 5 answer choice, only D support the argument, the other four answer choices weaken the argument. So, D is correct answer.

Master | Next Rank: 500 Posts
Posts: 126
Joined: Sat Sep 26, 2009 5:41 pm
Location: Chennai
Thanked: 4 times
Followed by:2 members
GMAT Score:660

by vivek1110 » Sat May 15, 2010 7:16 am
Let me give this a shot,

Stimulus states: DARPA projects initiative will help Industrial companies as well as the military; Often, military research has resulted in technology that has led to corporate development, and this pattern will continue to hold in the future.

A: Initiative occupies scientists who would have otherwise worked in corporates companies - Not Relevant to argument (Therefore might NOT Weaken it - Contender)

B: If the military refuses to share research, then companies will not be benefited - Weakens

C: If the money spent here would have served to benefit companies by giving them a tax rebate, then this initiative is actually hurting them. - Weakens.

D: States that this promotes nano tech research, which benefits corporates - Strengthens/Contender.

E: If the processes used cannot be used by industries, it would not benefit them. - Weakens

We're down to A & D.

D Actually strengthens the argument, as opposed to A, which does nothing to it.

Therefore D!
Is caught between a rock and a hard place!

Legendary Member
Posts: 809
Joined: Wed Mar 24, 2010 10:10 pm
Thanked: 50 times
Followed by:4 members

by akhpad » Wed May 19, 2010 4:22 am
Thanks